Algebraic Techniques 05 - Monotonicity and Floor Function

f:XXf:X \rightarrow X is non-empty subset of   R\; \mathbb{R} and   f[n](x)=x\; f^{[n]}(x) = x where   xX\; x \in X and   n1\; n \geq 1. Prove the following statements.

(a) f(x)=x if f is increasing(b) f(f(x))=x if f is decreasing and even f(x)=x if n is odd\begin{aligned} &\text{(a) } f(x) = x \text{ if } f \text{ is increasing} \\ &\text{(b) } f(f(x)) = x \text{ if } f \text{ is decreasing and even } f(x) = x \text{ if } n \text{ is odd} \end{aligned}

[Solution] (a) Assume that f(x)<xf(x) < x.Since ff is increasing, f(f(x))<f(x)f(f(x)) < f(x). Iterating this step, x=f[n](x)<f[n1](x)<<f(x)<xcontradiction\begin{aligned} x = f^{[n]}(x) < f^{[n - 1]}(x) < \cdots < f(x) < x \Longrightarrow \text{contradiction} \end{aligned}

In the same manner, assume that f(x)>xf(x) > x. Then x=f[n](x)>f[n1](x)>>f(x)>xcontradiction\begin{aligned} x = f^{[n]}(x) > f^{[n - 1]}(x) > \cdots > f(x) > x \Longrightarrow \text{contradiction} \end{aligned}

Therefore, f(x)=xf(x) = x.

(b) Note that f[2n](x)=f[n](f[n](x))=f[n](x)=xf^{[2n]}(x) = f^{[n]}(f^{[n]}(x)) = f^{[n]}(x) = x. Let g=ffg = f \circ f, then f[2n](x)=g[n](x)f^{[2n]}(x) = g^{[n]}(x). So, g[n](x)=xg^{[n]}(x) = x. However, gg is increasing because ff is decreasing. Applying (a) to gg, g(x)=xg(x) = x and f(f(x))=xf(f(x)) = x. Lastly, if nn is odd, f[n1](x)=xf^{[n - 1]}(x) = x because f(f(x))=xf(f(x)) = x and n1n - 1 is even. As a result, x=f[n](x)=f(f[n1](x))=f(x)x = f^{[n]}(x) = f(f^{[n - 1]}(x)) = f(x).

Find solutions of the following system of equations.

{3a=(b+c+d)33b=(c+d+a)33c=(d+a+b)33d=(a+b+c)3\begin{aligned} \begin{cases} 3a = (b + c + d)^3 \\ 3b = (c + d + a)^3 \\ 3c = (d + a + b)^3 \\ 3d = (a + b + c)^3 \end{cases} \end{aligned}

[Solution] Reforming the equations, {3a3=b+c+d3b3=c+d+a3c3=d+a+b3d3=a+b+c\begin{aligned} \begin{cases} \sqrt[3]{3a} = b + c + d \\ \sqrt[3]{3b} = c + d + a \\ \sqrt[3]{3c} = d + a + b \\ \sqrt[3]{3d} = a + b + c \end{cases} \end{aligned}

It implies that a+3a3=b+3b3=c+3c3=d+3d3=a+b+c+d\begin{aligned} a + \sqrt[3]{3a} = b + \sqrt[3]{3b} = c + \sqrt[3]{3c} = d + \sqrt[3]{3d} = a + b + c + d \end{aligned}

The key point is that the function xx+x3x \rightarrow x + \sqrt[3]{x} is injective and increasing because it is a sum of two increasing functions. Therefore, a=b=c=da = b = c = d. 3a=(3a)3=27a3a=0 or a=±13\begin{aligned} 3a = (3a)^3 = 27a^3 \Longrightarrow a = 0 \text{ or } a = \pm \frac{1}{3} \end{aligned}

Find solutions of the following system of equations.

{x4=4y3y4=4z3z4=4x3\begin{aligned} \begin{cases} x^4 = 4y - 3 \\ y^4 = 4z - 3 \\ z^4 = 4x - 3 \end{cases} \end{aligned}

[Solution] Reforming the equations, {y=x4+34z=y4+34x=z4+34\begin{aligned} \begin{cases} y = \frac{x^4 + 3}{4} \\ z = \frac{y^4 + 3}{4} \\ x = \frac{z^4 + 3}{4} \end{cases} \end{aligned}

It implies that x>0x > 0, y>0y > 0, and z>0z > 0. Let f(t)=(t4+3)/4f(t) = (t^4 + 3) / 4, then f(t)f(t) is increasing for t>0t > 0. The reformed system tells that it is cyclic. If ff is increasing and has cyclic system, it must be true that f(x)=xf(x) = x. Therefore, x=y=zx = y = z and f(t)=tf(t) = t, which means t4+3=4tt^4 + 3 = 4t. By AM-GM, t4+3=t4+1+1+14t\begin{aligned} t^4 + 3 = t^4 + 1 + 1 + 1 \geq 4t \end{aligned}

and the equal sign is valid when t=1t = 1. So, the solution is (x,y,z)=(1,1,1)(x, y, z) = (1, 1, 1).

Prove Hermite’s identity. That is, show that the following equation holds for any real number x\, x and any positive integer n\, n.

x+x+1n++x+n1n=nx\begin{aligned} \lfloor x \rfloor + \left\lfloor x + \frac{1}{n} \right\rfloor + \cdots + \left\lfloor x + \frac{n - 1}{n} \right\rfloor = \lfloor nx \rfloor \end{aligned}

[Solution] Let x=x+αx = \lfloor x \rfloor + \alpha for α[0,1)\alpha \in [0, 1). Using the fact that x+n=x+n\lfloor x + n \rfloor = \lfloor x \rfloor + n, x+x+1n++x+n1n=  x+x+α+1n++x+α+n1n=  nx+α+1n++α+n1nnx=nx+nα=nx+nα\begin{aligned} &\lfloor x \rfloor + \left\lfloor x + \frac{1}{n} \right\rfloor + \cdots + \left\lfloor x + \frac{n - 1}{n} \right\rfloor \\ = \; &\lfloor x \rfloor + \left\lfloor \lfloor x \rfloor + \alpha + \frac{1}{n} \right\rfloor + \cdots + \left\lfloor \lfloor x \rfloor + \alpha + \frac{n - 1}{n} \right\rfloor \\ = \; &n\lfloor x \rfloor + \left\lfloor \alpha + \frac{1}{n} \right\rfloor + \cdots + \left\lfloor \alpha + \frac{n - 1}{n} \right\rfloor \\\\\\ &\lfloor nx \rfloor = \lfloor n \lfloor x \rfloor + n \alpha \rfloor = n \lfloor x \rfloor + \lfloor n \alpha \rfloor \end{aligned}

Let k=nαk = \lfloor n \alpha \rfloor such that α[k/n,(k+1)/n)\alpha \in [k/n, (k + 1)/n), then knα<k+1k \leq n \alpha < k + 1 and k<nk < n since α<1\alpha < 1. Meanwhile, for j{1,2,,n1}j \in \{ 1, 2, \cdots, n - 1 \}, α+jn=0 or 1,α+jn=1    α+jn1    nαnj\begin{aligned} \left\lfloor \alpha + \frac{j}{n} \right\rfloor &= 0 \text{ or } 1, \\\\ \left\lfloor \alpha + \frac{j}{n} \right\rfloor = 1 \; \Longleftrightarrow \; &\alpha + \frac{j}{n} \geq 1 \; \Longleftrightarrow \; n \alpha \geq n - j \end{aligned}

It implies that α+j/n=1\left\lfloor \alpha + j/n \right\rfloor = 1 only when jnkj \geq n - k because nαnjn \alpha \geq n - j and nαkn \alpha \geq k for the integers kk and njn - j. Therefore, α+1n++α+n1n=j=nkn11=k=nα\begin{aligned} \left\lfloor \alpha + \frac{1}{n} \right\rfloor + \cdots + \left\lfloor \alpha + \frac{n - 1}{n} \right\rfloor = \sum_{j = n - k}^{n - 1}1 = k = \lfloor n \alpha \rfloor \end{aligned}

For all positive integer n\, n, prove the following equation.

n+2021++n+2n12n=n\begin{aligned} \left\lfloor \frac{n + 2^0}{2^1} \right\rfloor + \cdots + \left\lfloor \frac{n + 2^{n - 1}}{2^n} \right\rfloor = n \end{aligned}

[Solution] Reforming the equation, n2+12++n2n+12=n\begin{aligned} \left\lfloor \frac{n}{2} + \frac{1}{2} \right\rfloor + \cdots + \left\lfloor \frac{n}{2^n} + \frac{1}{2} \right\rfloor = n \end{aligned}

By Hermite’s identity, x+12=2xx\begin{aligned} \left\lfloor x + \frac{1}{2} \right\rfloor = \left\lfloor 2x \right\rfloor - \left\lfloor x \right\rfloor \end{aligned}

So, reformed equation can be rewritten as n2+12++n2n+12=nn2+n2n4++n2n1n2n=nn2n\begin{aligned} \left\lfloor \frac{n}{2} + \frac{1}{2} \right\rfloor + \cdots + \left\lfloor \frac{n}{2^n} + \frac{1}{2} \right\rfloor = n - \left\lfloor \frac{n}{2} \right\rfloor + \left\lfloor \frac{n}{2} \right\rfloor - \left\lfloor \frac{n}{4} \right\rfloor + \cdots + \left\lfloor \frac{n}{2^{n - 1}} \right\rfloor - \left\lfloor \frac{n}{2^n} \right\rfloor = n - \left\lfloor \frac{n}{2^n} \right\rfloor \end{aligned}

n/2n=0\left\lfloor n / 2^n \right\rfloor = 0 since 2n>n2^n > n for all nn, then the proof is complete.

For any real number x\, x and any positive integer n\, n, prove the following equation.

xn=xn\begin{aligned} \left\lfloor \frac{x}{n} \right\rfloor = \left\lfloor \frac{\lfloor x \rfloor}{n} \right\rfloor \end{aligned}

[Solution] Let k=x/nk = \lfloor x / n \rfloor, then x/n[k,k+1)x/n \in [k, k + 1), which means that x[nk,n(k+1))x \in [nk, n(k + 1)). Since floor function is non-decreasing and n(k+1)n(k + 1) is an integer, x[nk,n(k+1))\lfloor x \rfloor \in [nk, n(k + 1)). So, x/n[k,k+1)\lfloor x \rfloor / n \in [k, k + 1) and x/n=k\left\lfloor \lfloor x \rfloor / n \right\rfloor = k.

For all positive integer n\, n, prove the following equation.

n+n+1=4n+1\begin{aligned} \left\lfloor \sqrt{n} + \sqrt{n + 1} \right\rfloor = \left\lfloor \sqrt{4n + 1} \right\rfloor \end{aligned}

[Solution] Let 4n+1=k\left\lfloor \sqrt{4n + 1} \right\rfloor = k, then k24n+1<(k+1)2k^2 \leq 4n + 1 < (k + 1)^2. It is sufficient to prove that k2(n+n+1)2=2n+1+2n2+n<(k+1)2\begin{aligned} k^2 \leq \left( \sqrt{n} + \sqrt{n + 1} \right)^2 = 2n + 1 + 2 \sqrt{n^2 + n} < (k + 1)^2 \end{aligned}

The left side of this inequality is obvious because 2n+1+2n2+n>2n+1+2n=4n+1k2\begin{aligned} 2n + 1 + 2 \sqrt{n^2 + n} > 2n + 1 + 2n = 4n + 1 \geq k^2 \end{aligned}

In a similar manner, the right side is also true. 2n+1+2n2+n<2n+1+2(n+12)=4n+2(k+1)2\begin{aligned} 2n + 1 + 2 \sqrt{n^2 + n} < 2n + 1 + 2 \left( n + \frac{1}{2} \right) = 4n + 2 \leq (k + 1)^2 \end{aligned}

ana_n is a sequence of positive integers and is defined as follows. Find all positive integers of an\, a_n.

an=n+n+12\begin{aligned} a_n = \left\lfloor n + \sqrt{n} + \frac{1}{2} \right\rfloor \end{aligned}

[Solution] By observation, this sequence is increasing and skipping perfect squares as 22, 33, 55, 66, 77, 88, 1010, \cdots. The idea is to create new inequality to prove from these observations. Let an=ka_n = k, then this sequence skips k+1k + 1 only when an+1k+2a_{n + 1} \geq k + 2. That is, n+1+n+1+12n+1+n+1+12k+2\begin{aligned} n + 1 + \sqrt{n + 1} + \frac{1}{2} \geq \left\lfloor n + 1 + \sqrt{n + 1} + \frac{1}{2} \right\rfloor \geq k + 2 \end{aligned}

Let m=knm = k - n, then this inequality can be rewritten as n+1kn+12  n+1(m+12)2=m2+m+14  nm2+m\begin{aligned} &\sqrt{n + 1} \geq k - n + \frac{1}{2} \\\\ \Longrightarrow \; &n + 1 \geq \left( m + \frac{1}{2} \right)^2 = m^2 + m + \frac{1}{4} \\\\ \Longrightarrow \; &n \geq m^2 + m \end{aligned}

since mm and nn are integers. In addition, from an=ka_n = k, kn+n+12<k+1  (m12)2n<(m+12)2  m2m<nm2+m\begin{aligned} &k \leq n + \sqrt{n} + \frac{1}{2} < k + 1 \\\\ \Longrightarrow \; &\left( m - \frac{1}{2} \right)^2 \leq n < \left( m + \frac{1}{2} \right)^2 \\\\ \Longrightarrow \; &m^2 - m < n \leq m^2 + m \end{aligned}

since mm and nn are integers. Combining the previous condition, n=m2+mn = m^2 + m. So, k=n+m=m2+2mk = n + m = m^2 + 2m, which means k+1=(m+1)2k + 1 = (m + 1)^2 after adding 11 to both sides. Therefore, this sequence skips the positive integer k+1k + 1 only when k+1=(m+1)2k + 1 = (m + 1)^2 for all non-negative mm. In other words, the sequence ana_n skips an+1a_n + 1 only when an+1=(ann+1)2a_n + 1 = (a_n - n + 1)^2. Accordingly, ana_n is a subset of all positive integers except for perfect squares.

Reference

[1] Titu Andreescu, 105 Algebra Problems.


© 2024. All rights reserved.